Find Distribution of Sum of Random Variables given only Joint Distribution












0












$begingroup$


So if we have two random variables $X, Y$, with unknown distributions. Then we have a random variable $Z$, such that $Z=X+Y$.



Firstly, how do we find the CDF of $Z$, i.e. $F_Z(z)$, given the joint PDF of $X, Y$;



$$f_{XY}(x,y) for x, y > 0$$



Intuitively, as far as I can tell, the expression to find this is (hopefully):



$$F_Z(z) = int_{0}^{z} int_{0}^{z-x} f_{XY}(x,y) dy dx $$



Except I wasn't certain about whether this is the case, so I wanted to prove it. Therefore my real question is how to show this expression is correct?



Perhaps something along the lines of ?:



begin{align}
F_Z(z) & = P(Z le z) \
& = P(X + Y le z) \
...??? \
...??? \
& = P(Y le z -x and X le z)
end{align}



Any help would be great!










share|cite|improve this question









$endgroup$












  • $begingroup$
    Your formula for the CDF of $Z$ is correct. You wrote $F_Z(z) = P(X+Y leq z)$. Can you express the last quantity in terms of the density?
    $endgroup$
    – Michh
    Dec 29 '18 at 16:34










  • $begingroup$
    @Michh I'm sorry, honestly, I've no clue. Any help?
    $endgroup$
    – ptolemy0
    Jan 1 at 10:08
















0












$begingroup$


So if we have two random variables $X, Y$, with unknown distributions. Then we have a random variable $Z$, such that $Z=X+Y$.



Firstly, how do we find the CDF of $Z$, i.e. $F_Z(z)$, given the joint PDF of $X, Y$;



$$f_{XY}(x,y) for x, y > 0$$



Intuitively, as far as I can tell, the expression to find this is (hopefully):



$$F_Z(z) = int_{0}^{z} int_{0}^{z-x} f_{XY}(x,y) dy dx $$



Except I wasn't certain about whether this is the case, so I wanted to prove it. Therefore my real question is how to show this expression is correct?



Perhaps something along the lines of ?:



begin{align}
F_Z(z) & = P(Z le z) \
& = P(X + Y le z) \
...??? \
...??? \
& = P(Y le z -x and X le z)
end{align}



Any help would be great!










share|cite|improve this question









$endgroup$












  • $begingroup$
    Your formula for the CDF of $Z$ is correct. You wrote $F_Z(z) = P(X+Y leq z)$. Can you express the last quantity in terms of the density?
    $endgroup$
    – Michh
    Dec 29 '18 at 16:34










  • $begingroup$
    @Michh I'm sorry, honestly, I've no clue. Any help?
    $endgroup$
    – ptolemy0
    Jan 1 at 10:08














0












0








0





$begingroup$


So if we have two random variables $X, Y$, with unknown distributions. Then we have a random variable $Z$, such that $Z=X+Y$.



Firstly, how do we find the CDF of $Z$, i.e. $F_Z(z)$, given the joint PDF of $X, Y$;



$$f_{XY}(x,y) for x, y > 0$$



Intuitively, as far as I can tell, the expression to find this is (hopefully):



$$F_Z(z) = int_{0}^{z} int_{0}^{z-x} f_{XY}(x,y) dy dx $$



Except I wasn't certain about whether this is the case, so I wanted to prove it. Therefore my real question is how to show this expression is correct?



Perhaps something along the lines of ?:



begin{align}
F_Z(z) & = P(Z le z) \
& = P(X + Y le z) \
...??? \
...??? \
& = P(Y le z -x and X le z)
end{align}



Any help would be great!










share|cite|improve this question









$endgroup$




So if we have two random variables $X, Y$, with unknown distributions. Then we have a random variable $Z$, such that $Z=X+Y$.



Firstly, how do we find the CDF of $Z$, i.e. $F_Z(z)$, given the joint PDF of $X, Y$;



$$f_{XY}(x,y) for x, y > 0$$



Intuitively, as far as I can tell, the expression to find this is (hopefully):



$$F_Z(z) = int_{0}^{z} int_{0}^{z-x} f_{XY}(x,y) dy dx $$



Except I wasn't certain about whether this is the case, so I wanted to prove it. Therefore my real question is how to show this expression is correct?



Perhaps something along the lines of ?:



begin{align}
F_Z(z) & = P(Z le z) \
& = P(X + Y le z) \
...??? \
...??? \
& = P(Y le z -x and X le z)
end{align}



Any help would be great!







statistics probability-distributions bivariate-distributions






share|cite|improve this question













share|cite|improve this question











share|cite|improve this question




share|cite|improve this question










asked Dec 29 '18 at 16:13









ptolemy0ptolemy0

154




154












  • $begingroup$
    Your formula for the CDF of $Z$ is correct. You wrote $F_Z(z) = P(X+Y leq z)$. Can you express the last quantity in terms of the density?
    $endgroup$
    – Michh
    Dec 29 '18 at 16:34










  • $begingroup$
    @Michh I'm sorry, honestly, I've no clue. Any help?
    $endgroup$
    – ptolemy0
    Jan 1 at 10:08


















  • $begingroup$
    Your formula for the CDF of $Z$ is correct. You wrote $F_Z(z) = P(X+Y leq z)$. Can you express the last quantity in terms of the density?
    $endgroup$
    – Michh
    Dec 29 '18 at 16:34










  • $begingroup$
    @Michh I'm sorry, honestly, I've no clue. Any help?
    $endgroup$
    – ptolemy0
    Jan 1 at 10:08
















$begingroup$
Your formula for the CDF of $Z$ is correct. You wrote $F_Z(z) = P(X+Y leq z)$. Can you express the last quantity in terms of the density?
$endgroup$
– Michh
Dec 29 '18 at 16:34




$begingroup$
Your formula for the CDF of $Z$ is correct. You wrote $F_Z(z) = P(X+Y leq z)$. Can you express the last quantity in terms of the density?
$endgroup$
– Michh
Dec 29 '18 at 16:34












$begingroup$
@Michh I'm sorry, honestly, I've no clue. Any help?
$endgroup$
– ptolemy0
Jan 1 at 10:08




$begingroup$
@Michh I'm sorry, honestly, I've no clue. Any help?
$endgroup$
– ptolemy0
Jan 1 at 10:08










1 Answer
1






active

oldest

votes


















0












$begingroup$

We have
$$begin{align*} F_Z(z) &= P(X+Y leq z) = E[mathbf{1}_{X+Y leq z}]\
&= int_{[0,infty) times [0,infty)} mathbf{1}_{x+y leq z} ; f_{X,Y}(x,y) , mathrm{d}x mathrm{d}y
end{align*}$$

using the following property of the density: $E[g(X,Y)] = int_{[0,infty) times [0,infty)} g(x,y), f_{X,Y}(x,y) , mathrm{d}x mathrm{d}y$. Now notice that $mathbf{1}_{x+y leq z} = mathbf{1}_{x leq z} mathbf{1}_{y leq z - x}$, that is $x+y leq z$ if and only if $x leq z$ and $y leq z - x$. So using Fubini's theorem, we get
$$
begin{align*} F_Z(z) = int_{[0,infty) times [0,infty)} mathbf{1}_{x leq z} mathbf{1}_{y leq z - x} ; f_{X,Y}(x,y) , mathrm{d}x mathrm{d}y &= int_{[0,infty)} mathbf{1}_{x leq z} left(int_{[0,infty)}mathbf{1}_{y leq z - x} ; f_{X,Y}(x,y) , mathrm{d}y right)mathrm{d}x \
&= int_0^z left(int_0^{z-x} f_{X,Y}(x,y) , mathrm{d}y right)mathrm{d}x
end{align*}$$

which proves the result. Normally you would not have to go into as much detail and from the first equation above you could directly obtain the result.






share|cite|improve this answer









$endgroup$













    Your Answer





    StackExchange.ifUsing("editor", function () {
    return StackExchange.using("mathjaxEditing", function () {
    StackExchange.MarkdownEditor.creationCallbacks.add(function (editor, postfix) {
    StackExchange.mathjaxEditing.prepareWmdForMathJax(editor, postfix, [["$", "$"], ["\\(","\\)"]]);
    });
    });
    }, "mathjax-editing");

    StackExchange.ready(function() {
    var channelOptions = {
    tags: "".split(" "),
    id: "69"
    };
    initTagRenderer("".split(" "), "".split(" "), channelOptions);

    StackExchange.using("externalEditor", function() {
    // Have to fire editor after snippets, if snippets enabled
    if (StackExchange.settings.snippets.snippetsEnabled) {
    StackExchange.using("snippets", function() {
    createEditor();
    });
    }
    else {
    createEditor();
    }
    });

    function createEditor() {
    StackExchange.prepareEditor({
    heartbeatType: 'answer',
    autoActivateHeartbeat: false,
    convertImagesToLinks: true,
    noModals: true,
    showLowRepImageUploadWarning: true,
    reputationToPostImages: 10,
    bindNavPrevention: true,
    postfix: "",
    imageUploader: {
    brandingHtml: "Powered by u003ca class="icon-imgur-white" href="https://imgur.com/"u003eu003c/au003e",
    contentPolicyHtml: "User contributions licensed under u003ca href="https://creativecommons.org/licenses/by-sa/3.0/"u003ecc by-sa 3.0 with attribution requiredu003c/au003e u003ca href="https://stackoverflow.com/legal/content-policy"u003e(content policy)u003c/au003e",
    allowUrls: true
    },
    noCode: true, onDemand: true,
    discardSelector: ".discard-answer"
    ,immediatelyShowMarkdownHelp:true
    });


    }
    });














    draft saved

    draft discarded


















    StackExchange.ready(
    function () {
    StackExchange.openid.initPostLogin('.new-post-login', 'https%3a%2f%2fmath.stackexchange.com%2fquestions%2f3055990%2ffind-distribution-of-sum-of-random-variables-given-only-joint-distribution%23new-answer', 'question_page');
    }
    );

    Post as a guest















    Required, but never shown

























    1 Answer
    1






    active

    oldest

    votes








    1 Answer
    1






    active

    oldest

    votes









    active

    oldest

    votes






    active

    oldest

    votes









    0












    $begingroup$

    We have
    $$begin{align*} F_Z(z) &= P(X+Y leq z) = E[mathbf{1}_{X+Y leq z}]\
    &= int_{[0,infty) times [0,infty)} mathbf{1}_{x+y leq z} ; f_{X,Y}(x,y) , mathrm{d}x mathrm{d}y
    end{align*}$$

    using the following property of the density: $E[g(X,Y)] = int_{[0,infty) times [0,infty)} g(x,y), f_{X,Y}(x,y) , mathrm{d}x mathrm{d}y$. Now notice that $mathbf{1}_{x+y leq z} = mathbf{1}_{x leq z} mathbf{1}_{y leq z - x}$, that is $x+y leq z$ if and only if $x leq z$ and $y leq z - x$. So using Fubini's theorem, we get
    $$
    begin{align*} F_Z(z) = int_{[0,infty) times [0,infty)} mathbf{1}_{x leq z} mathbf{1}_{y leq z - x} ; f_{X,Y}(x,y) , mathrm{d}x mathrm{d}y &= int_{[0,infty)} mathbf{1}_{x leq z} left(int_{[0,infty)}mathbf{1}_{y leq z - x} ; f_{X,Y}(x,y) , mathrm{d}y right)mathrm{d}x \
    &= int_0^z left(int_0^{z-x} f_{X,Y}(x,y) , mathrm{d}y right)mathrm{d}x
    end{align*}$$

    which proves the result. Normally you would not have to go into as much detail and from the first equation above you could directly obtain the result.






    share|cite|improve this answer









    $endgroup$


















      0












      $begingroup$

      We have
      $$begin{align*} F_Z(z) &= P(X+Y leq z) = E[mathbf{1}_{X+Y leq z}]\
      &= int_{[0,infty) times [0,infty)} mathbf{1}_{x+y leq z} ; f_{X,Y}(x,y) , mathrm{d}x mathrm{d}y
      end{align*}$$

      using the following property of the density: $E[g(X,Y)] = int_{[0,infty) times [0,infty)} g(x,y), f_{X,Y}(x,y) , mathrm{d}x mathrm{d}y$. Now notice that $mathbf{1}_{x+y leq z} = mathbf{1}_{x leq z} mathbf{1}_{y leq z - x}$, that is $x+y leq z$ if and only if $x leq z$ and $y leq z - x$. So using Fubini's theorem, we get
      $$
      begin{align*} F_Z(z) = int_{[0,infty) times [0,infty)} mathbf{1}_{x leq z} mathbf{1}_{y leq z - x} ; f_{X,Y}(x,y) , mathrm{d}x mathrm{d}y &= int_{[0,infty)} mathbf{1}_{x leq z} left(int_{[0,infty)}mathbf{1}_{y leq z - x} ; f_{X,Y}(x,y) , mathrm{d}y right)mathrm{d}x \
      &= int_0^z left(int_0^{z-x} f_{X,Y}(x,y) , mathrm{d}y right)mathrm{d}x
      end{align*}$$

      which proves the result. Normally you would not have to go into as much detail and from the first equation above you could directly obtain the result.






      share|cite|improve this answer









      $endgroup$
















        0












        0








        0





        $begingroup$

        We have
        $$begin{align*} F_Z(z) &= P(X+Y leq z) = E[mathbf{1}_{X+Y leq z}]\
        &= int_{[0,infty) times [0,infty)} mathbf{1}_{x+y leq z} ; f_{X,Y}(x,y) , mathrm{d}x mathrm{d}y
        end{align*}$$

        using the following property of the density: $E[g(X,Y)] = int_{[0,infty) times [0,infty)} g(x,y), f_{X,Y}(x,y) , mathrm{d}x mathrm{d}y$. Now notice that $mathbf{1}_{x+y leq z} = mathbf{1}_{x leq z} mathbf{1}_{y leq z - x}$, that is $x+y leq z$ if and only if $x leq z$ and $y leq z - x$. So using Fubini's theorem, we get
        $$
        begin{align*} F_Z(z) = int_{[0,infty) times [0,infty)} mathbf{1}_{x leq z} mathbf{1}_{y leq z - x} ; f_{X,Y}(x,y) , mathrm{d}x mathrm{d}y &= int_{[0,infty)} mathbf{1}_{x leq z} left(int_{[0,infty)}mathbf{1}_{y leq z - x} ; f_{X,Y}(x,y) , mathrm{d}y right)mathrm{d}x \
        &= int_0^z left(int_0^{z-x} f_{X,Y}(x,y) , mathrm{d}y right)mathrm{d}x
        end{align*}$$

        which proves the result. Normally you would not have to go into as much detail and from the first equation above you could directly obtain the result.






        share|cite|improve this answer









        $endgroup$



        We have
        $$begin{align*} F_Z(z) &= P(X+Y leq z) = E[mathbf{1}_{X+Y leq z}]\
        &= int_{[0,infty) times [0,infty)} mathbf{1}_{x+y leq z} ; f_{X,Y}(x,y) , mathrm{d}x mathrm{d}y
        end{align*}$$

        using the following property of the density: $E[g(X,Y)] = int_{[0,infty) times [0,infty)} g(x,y), f_{X,Y}(x,y) , mathrm{d}x mathrm{d}y$. Now notice that $mathbf{1}_{x+y leq z} = mathbf{1}_{x leq z} mathbf{1}_{y leq z - x}$, that is $x+y leq z$ if and only if $x leq z$ and $y leq z - x$. So using Fubini's theorem, we get
        $$
        begin{align*} F_Z(z) = int_{[0,infty) times [0,infty)} mathbf{1}_{x leq z} mathbf{1}_{y leq z - x} ; f_{X,Y}(x,y) , mathrm{d}x mathrm{d}y &= int_{[0,infty)} mathbf{1}_{x leq z} left(int_{[0,infty)}mathbf{1}_{y leq z - x} ; f_{X,Y}(x,y) , mathrm{d}y right)mathrm{d}x \
        &= int_0^z left(int_0^{z-x} f_{X,Y}(x,y) , mathrm{d}y right)mathrm{d}x
        end{align*}$$

        which proves the result. Normally you would not have to go into as much detail and from the first equation above you could directly obtain the result.







        share|cite|improve this answer












        share|cite|improve this answer



        share|cite|improve this answer










        answered Jan 1 at 10:46









        MichhMichh

        22316




        22316






























            draft saved

            draft discarded




















































            Thanks for contributing an answer to Mathematics Stack Exchange!


            • Please be sure to answer the question. Provide details and share your research!

            But avoid



            • Asking for help, clarification, or responding to other answers.

            • Making statements based on opinion; back them up with references or personal experience.


            Use MathJax to format equations. MathJax reference.


            To learn more, see our tips on writing great answers.




            draft saved


            draft discarded














            StackExchange.ready(
            function () {
            StackExchange.openid.initPostLogin('.new-post-login', 'https%3a%2f%2fmath.stackexchange.com%2fquestions%2f3055990%2ffind-distribution-of-sum-of-random-variables-given-only-joint-distribution%23new-answer', 'question_page');
            }
            );

            Post as a guest















            Required, but never shown





















































            Required, but never shown














            Required, but never shown












            Required, but never shown







            Required, but never shown

































            Required, but never shown














            Required, but never shown












            Required, but never shown







            Required, but never shown







            Popular posts from this blog

            Human spaceflight

            Can not write log (Is /dev/pts mounted?) - openpty in Ubuntu-on-Windows?

            File:DeusFollowingSea.jpg